LSAT and Law School Admissions Forum

Get expert LSAT preparation and law school admissions advice from PowerScore Test Preparation.

 Administrator
PowerScore Staff
  • PowerScore Staff
  • Posts: 8937
  • Joined: Feb 02, 2011
|
#64125
Complete Question Explanation

Resolve the Paradox. The correct answer choice is (E)

This passage presents the results of a study of the relationship between salt intake and high blood
pressure. Heavy salt intake tends to increase blood pressure. Predictably, many high blood pressure
sufferers experienced a blood pressure decrease when they cut their salt intake. There were also
participants, however, who had heavy salt intake throughout the study and yet were able to somehow
maintain low blood-pressure.

The passage is followed by a Resolve question stem, so the correct answer choice will provide some
explanation of those who, contrary to normal expectations, had high salt intake throughout and
maintained low blood pressure regardless.

Answer choice (A): The study participants who create the paradox are those who had low blood
pressure despite maintaining high levels of salt intake throughout the study. This choice deals with a
different group, and does not resolve the discrepancy.

Answer choice (B): Even if there are other factors associated with high blood pressure, this does
not explain the fact that, despite a general correlation between heavy salt intake and higher blood
pressure, some participants had consistently high salt intake and consistently low blood pressure.

Answer choice (C): Just because salt reduction is not the best way to reduce high blood pressure, salt
reduction is still one approach that we would expect to be effective. We would still expect those who
eat a lot of salt generally to have higher blood pressure, so this choice fails to resolve the paradox.

Answer choice (D): Much like incorrect answer choice (A) above, this choice deals with the wrong
population. The author presents the paradoxical case of study participants who ate a lot of salt and
still maintained high blood pressure. This choice deals with the opposite: those who have low salt
intake and high blood pressure. If anything this would actually broaden the apparent discrepancy,
making the study results more puzzling, so it cannot be the correct answer choice.

Answer choice (E): This is the correct answer choice. This choice describes a group who
could have low blood pressure despite high salt intake. This is a group of people who suffer from
abnormally low blood pressure, and crave a lot of salt just to keep their blood pressure levels high
enough. For such people, we might expect the otherwise paradoxical results from the study.
 moshei24
  • Posts: 465
  • Joined: Mar 20, 2012
|
#5429
Is the reason that the answer is E and not B because E contributes more than B does? Meaning, if E wasn't there, would B be the correct answer? It's funny - 46% picked each answer, and to have the breakdown is EXTREMELY rare.

Thanks!
 Adam Tyson
PowerScore Staff
  • PowerScore Staff
  • Posts: 5271
  • Joined: Apr 14, 2011
|
#5448
I think B is an attractive answer, at first, and I can understand why a lot of folks would choose it, but in the end it comes down to this: when resolving a paradox, you need to actively address both sides of the paradox, and answer B really doesn't address the second situation in our stimulus (that some people with high salt intake have low BP). It might work if you help it, by adding to the answer "well, if there are other factors, maybe some of them counteract the salt and lower BP." Resist the urge to "help" the answers - when you do, you are falling into a common trap set by the authors of the test. These folks are trying to trick and trap you, so why help them by giving them a hand and mentally improving their answers? Take them at face value - when you do that, B really has nothing to offer, and E has a much clearer, more active resolution of the paradox.

That said, if E was removed and you had no better answer in its place, B would be the best of what's left. The credited response isn't always a good answer - it's the best one that we have to choose from.

Adam
 moshei24
  • Posts: 465
  • Joined: Mar 20, 2012
|
#5452
Gotcha. Thank you!
 Basia W
  • Posts: 108
  • Joined: Jun 19, 2014
|
#16282
Good evening,

with regards to this question and its answer choice, although E explains an active resolution of one group of participants I feel like it ignores the other side of the paradox- thereby only explaining only one side of the paradox. Is this all right because it is addressing the conclusion specifically?

Thanks,

Basia
 Ron Gore
PowerScore Staff
  • PowerScore Staff
  • Posts: 220
  • Joined: May 15, 2013
|
#16298
Hi Basia,

First, please understand that a Paradox question does not contain an argument, so it will not have a conclusion.

Next, the paradox in this stimulus is that some people with high salt intake had very low blood pressure, even though it is the case that high salt intake is known to increase blood pressure. Our prephrase is that the correct answer choice will be something that causes some people with high salt intake to be different than the norm, to have low blood pressure. In other words, we are looking for something that makes these people different than the norm.

Answer choice (E) does this by telling us that those people had abnormally low blood pressure to begin with and had heightened salt cravings, which makes them different than the norm in a way that is relevant to salt intake and blood pressure. So, this answer choice does in fact address both side of the paradox in a way that helps to resolve it.

Please let me know if I can be of further help.

Thanks,

Ron
 Basia W
  • Posts: 108
  • Joined: Jun 19, 2014
|
#16317
Hello,

I forgot- Must be True, Paradox and Cannot Be True's have no conclusions.

As for your answer- it clarified it very well.

Thank you again!

Best,

Basia
 dyogenes
  • Posts: 7
  • Joined: Mar 11, 2019
|
#65426
Perhaps I just ended up on the wrong side of semantic intuition here, but using the "help" metric in the post above, I seem to arrive at an impasse.

Though it is true that we have to help B a bit (grant that at least one of the factors is limiting to salt's trend of increasing BP), I also find myself helping E to a similar degree. I found that E did not provide a sufficient connection between heightened salt cravings and very high salt intake.

E connects heightened salt cravings to abnormally low BP. But not only do we have to help out the answer by assuming salt cravings are proportional salt intake, we also have to help it out by assuming that heightened salt cravings get such people all the the way to a "very high salt intake" from wherever they begin (as opposed to raising salt intake to perhaps a normal level, or an elevated but not very high level). Perhaps people suffering from abnormally low BP have abnormally low salt intake, and it is raised to normal levels. Or perhaps such people have a low to normal salt intake, and it is raised to somewhere short of very high. It seems reasonable to assume that heightened cravings increase salt intake (relative to where they were), yet it seems fallacious to assume heightened cravings increase intake to any specific point - especially a very high point.

The LSAT makes jumps similar to the 'craving' to intake' jump found here somewhat frequently, so to me that by itself wasn't a defeater to answer E. However, it seems there's also a nontrivial assumption about the threshold/degree to which salt intake is increased by those with heightened salt cravings from abnormally low BP.

Also, the account of the help we have to give B for it to work seems to me to be too severe. It doesn't seem to me that we have to stipulate that among the other factors there are some that "counteract the salt and lower the BP". We know both from the first sentence of the passage (and found implicit in the correct answer choice E, as well) that very high salt intake alone is not sufficient for high BP. We only know that it tends to increase it, and that in those with already high BP that a reduction in salt intake reduces it. Consequently, all we must stipulate is that any one of the other dietary factors associated with high BP also increases BP (so by lacking such a factor, people might have a very high salt intake without a very high BP).

E lays out a potential situation for accounting for very high salt intake and very low BP, but then again B does the same. I'm having trouble seeing the advantage to the jumps we have to make in E over the ones in B. Is there a principled way to navigate this that I'm just not seeing?

Thank you!
 George George
PowerScore Staff
  • PowerScore Staff
  • Posts: 48
  • Joined: Jun 07, 2019
|
#65464
Very good commentary, here, Dyogenes. I also was initially very tempted by (B), but chose (E). Here's perhaps a different way of looking at it.

Answer (B) is vague. Sure, it suggests that there *might* be some other factor at work, but it doesn't suggest that there *is* some other factor at work. In other words, (B) sort-of "sits the fence." It says it's possible for some other "dietary factor" to come into play, but that alone is insufficient to tell you that that factor *is* at work in the high-salty low-BP people. (These kinds of answers are often *quite tempting*, imho, on Resolve the Paradox Qs as well as Strengthen Qs and Weaken Qs. But I have *never* seen one be the credited response on those Q-types.) In other words, answers which suggest only that there are other possible factors, without more, are usually too weak to have the kind of effect the Q requires.

Answer (E), on the other hand, goes further than suggesting there are other possibilities. (E) states those will low BP crave more salt to prevent their BP from dropping. Although cravings do not per se equate to a "very high salt intake" (I concur with this), cravings are relevant to salt intake because normally people will satisfy their gustatory cravings (if they're able, and there seems to be no reason to assume people in a study are being deprived of food). In other words, the increased cravings provide a reason to think that there might be a subset of people in the study who maintain "very low blood pressure" but still have a "very high salt intake," namely, because they are people predisposed to low BP, who happen to crave salt as a fix.

So, whereas (B) only brings up the possibility of other "dietary factors," without spelling out how they would impact this study, (E) targets the specific anomaly of the low BP/high salt subgroup. Where (B) remains vague, (E) is "on-point."

Remember finally that *even if* the LSAT were to write 2 answers, the Q-stem gives them a loophole by stating "contributes the *most* to an explanation." So, based on that fallback provision, the credited response would still be (E).
 dyogenes
  • Posts: 7
  • Joined: Mar 11, 2019
|
#65473
Thank you for the great reply! Reading it has helped me come to be able to dissect the psychology of the question-design a bit more in it's regarding B) to be wrong. So functionally I think the issue is resolved and that I'll be better able to navigate the rare question like this in the future. Will have to retrain myself to consider gaps like the one between "cravings" and "intake" to be less egregious.

Thanks!

Get the most out of your LSAT Prep Plus subscription.

Analyze and track your performance with our Testing and Analytics Package.